Last visit was: 04 Jun 2024, 20:41 It is currently 04 Jun 2024, 20:41
Close
GMAT Club Daily Prep
Thank you for using the timer - this advanced tool can estimate your performance and suggest more practice questions. We have subscribed you to Daily Prep Questions via email.

Customized
for You

we will pick new questions that match your level based on your Timer History

Track
Your Progress

every week, we’ll send you an estimated GMAT score based on your performance

Practice
Pays

we will pick new questions that match your level based on your Timer History
Not interested in getting valuable practice questions and articles delivered to your email? No problem, unsubscribe here.
Close
Request Expert Reply
Confirm Cancel
SORT BY:
Date
Tags:
Difficulty: 505-555 Level,    Weaken,                            
Show Tags
Hide Tags
User avatar
Manager
Manager
Joined: 08 Sep 2010
Posts: 113
Own Kudos [?]: 752 [143]
Given Kudos: 21
Location: India
Concentration: Finance
 Q49  V19 GMAT 2: 620  Q44  V31
WE 1: 6 Year, Telecom(GSM)
Send PM
Most Helpful Reply
User avatar
Intern
Intern
Joined: 14 Apr 2010
Posts: 19
Own Kudos [?]: 72 [56]
Given Kudos: 1
Send PM
User avatar
Manhattan Prep Instructor
Joined: 08 May 2012
Posts: 51
Own Kudos [?]: 902 [30]
Given Kudos: 4
Location: United States
GMAT Focus 1:
775 Q90 V90 DI86
General Discussion
User avatar
Senior Manager
Senior Manager
Joined: 24 Jun 2010
Status:Time to step up the tempo
Posts: 273
Own Kudos [?]: 675 [12]
Given Kudos: 50
Location: Milky way
Concentration: International Business, Marketing
Schools:ISB, Tepper - CMU, Chicago Booth, LSB
Send PM
Re: Journalist: In physics journals, the number of articles reporting the [#permalink]
9
Kudos
3
Bookmarks
ankitranjan wrote:
Journalist: In physics journals, the number of articles reporting the results of experiments involving particle
accelerators was lower last year than it had been in previous years. Several of the particle accelerators at major
research institutions were out of service the year before last for repairs, so it is likely that the low number of
articles was due to the decline in availability of particle accelerators.
Which of the following, if true, most seriously undermines the journalist’s argument?
(A) Every article based on experiments with particle accelerators that was submitted for publication last year
actually was published.
Neutral. This option neither strengthens or weakens.
(B) The average time scientists must wait for access to a particle accelerator has declined over the last
several years.
Possible candidate.
(C) The number of physics journals was the same last year as in previous years.
Out of scope. Does not talk about the articles about the experiments involving particle accelerator.
(D) Particle accelerators can be used for more than one group of experiments in any given year.
New information. Out of scope.
(E) Recent changes in the editorial policies of several physics journals have decreased the likelihood that
articles concerning particle-accelerator research will be accepted for publication.
Possible candidate.
OA will be given tomorrow.


If You Like the question Consider KUDOS


Narrowed down to B and E.

Option B: The average wait time has declined over the past several years but it does not reflect the availability of the particle accelerator. Also even if we assume that the decreased wait time means high availability there could be other reasons for the low number of articles published.

Option E: Brings in another angle to the argument by noting that the editorial policy change would reduce the number of articles accepted for publication. Even though "recent" is not clearly defined this option is the most promising one.

My final answer is E.
avatar
Intern
Intern
Joined: 14 Aug 2010
Posts: 10
Own Kudos [?]: 29 [1]
Given Kudos: 3
Send PM
Re: Journalist: In physics journals, the number of articles reporting the [#permalink]
1
Kudos
ankitranjan wrote:
Journalist: In physics journals, the number of articles reporting the results of experiments involving particle
accelerators was lower last year than it had been in previous years. Several of the particle accelerators at major
research institutions were out of service the year before last for repairs, so it is likely that the low number of
articles was due to the decline in availability of particle accelerators.
Which of the following, if true, most seriously undermines the journalist’s argument?
(A) Every article based on experiments with particle accelerators that was submitted for publication last year actually was published. - Irrelevant.
(B) The average time scientists must wait for access to a particle accelerator has declined over the last several years. - doesnt address the conclusion.
(C) The number of physics journals was the same last year as in previous years. - Irrelevant
(D) Particle accelerators can be used for more than one group of experiments in any given year. irrelevant
(E) Recent changes in the editorial policies of several physics journals have decreased the likelihood that articles concerning particle-accelerator research will be accepted for publication. - Correct. This one will mean that there will be less articles on Particle accelerator.OA will be given tomorrow.


If You Like the question Consider KUDOS
avatar
Intern
Intern
Joined: 29 Mar 2011
Posts: 4
Own Kudos [?]: 7 [6]
Given Kudos: 3
Send PM
Re: Journalist: In physics journals, the number of articles reporting the [#permalink]
6
Kudos
What's wrong with D.
Both D and E are equal contenders.Both of them are weakning the argument in their own ways
D is weakening the argument by stating that
"Although the effect occurs, the cause was not the reason for the occurence of the effect"

E is weakening the argument by stating that
"The stated cause was not the reason that the effect occured"

How do you break the tie between the two, as both of them qualify equally in weakening the argument.
I find D more appealing in that, E mentions the word recent, which may mean different for different people.
It may mean this year, or it may mean previous 5 years. Who knows ? :)

Somebody kindly enlighten.
User avatar
Intern
Intern
Joined: 06 Jul 2011
Posts: 20
Own Kudos [?]: 50 [3]
Given Kudos: 0
Send PM
Re: Journalist: In physics journals, the number of articles reporting the [#permalink]
3
Kudos
suyash wrote:
What's wrong with D.
Both D and E are equal contenders.Both of them are weakning the argument in their own ways
D is weakening the argument by stating that
"Although the effect occurs, the cause was not the reason for the occurence of the effect"

E is weakening the argument by stating that
"The stated cause was not the reason that the effect occured"

How do you break the tie between the two, as both of them qualify equally in weakening the argument.
I find D more appealing in that, E mentions the word recent, which may mean different for different people.
It may mean this year, or it may mean previous 5 years. Who knows ? :)

Somebody kindly enlighten.


My question, as well. I chose D. The argument says the number of accelerators is directly proportional to the number of articles. But what if you can just double up the groups of experiments on the available accelerators. Then you can produce the same or more output in terms of articles.

Looking back, E is, however, a very strong answer. And if I did the question again, the tie break would be the sheer strength of that answer.

I guess the simplest explanation is most likely the correct one. And I have a bad habit of skipping over the more straightforward answers, over thinking, and picking the wrong ones. :|
User avatar
Senior Manager
Senior Manager
Joined: 23 Apr 2010
Posts: 476
Own Kudos [?]: 358 [2]
Given Kudos: 7
Send PM
Re: Journalist: In physics journals, the number of articles reporting the [#permalink]
2
Kudos
I also have had a hard time deciding between between (D) and (E).

(D) If particle accelerators can be used for more than one group of experiments in any given year, then the drop of the availability of accelerators due to their repairs could be offset by using available accelerators more.

(E) I dismissed this choice for two reasons:

1) The stimulus talks about articles reporting results of experiments involving particle accelerators; whereas (E) talks about articles concerning patricle-accelerator research, which IMO is not the same. And the reason I think these two are not the same is because particle accelerator research IMO studies accelerators, and experiments involving particle accelerators are concerned with behavior of particles in accelerators. E.g., (E) talks about CERN (European Laboratory for Particle Physics), stimulus talks about a research at one of CERN's accelerators.

2) Additionally, (E) talks about recent changes in editorial policies. The stimulus talks about events that are two years old.

Maybe I'm overthinking this one too much and maybe both underlined phrases are just the same. Maybe recent means two years ago. But anyway, I strongly think that (D) is a valid answer and not anywhere inferior to (E).

Here's OG's explanation (BTW it's Q83 from OG12):

(D) If the accelerators can be used for multiple experiments, then it is reasonable to expect more articles related to them, not fewer.

Now, I don't know why we can infer that. Since some accelerators were out of service, then it is reasonable to assume that even though accelerators can be used to process more than one experiment, the workload on working accelerators would increase thereby not allowing the overall number of experiments to increase.

Any thoughts?
User avatar
Senior Manager
Senior Manager
Joined: 23 Apr 2010
Posts: 476
Own Kudos [?]: 358 [3]
Given Kudos: 7
Send PM
Re: Journalist: In physics journals, the number of articles reporting the [#permalink]
2
Kudos
1
Bookmarks
For those who are interested, here's why (D) is a wrong answer:

The effect: There were fewer articles reporting the results of experiments involving particle accelerators.

The stated cause (the one we should attack): Some accelerators were unavailable for research.

To refute the cause and effect relationship we need to, for example, find an alternative cause.

(D) doesn't accomplish this because had it been true that other functioning accelerators were available for multiple experiments, the effect would have never occurred in the first place. Well of course if those accelerators hadn't been already used at their full capacities. But even if that were the case, (D) still wouldn't be a great alternative cause of the decline in the number of published articles.

(E) provides an alternative explanation of the stated effect.

The only thing that still bothers me is the word 'lately' in (E). But all in all (E) is a clear winner now.
avatar
Intern
Intern
Joined: 28 Feb 2011
Posts: 29
Own Kudos [?]: 71 [1]
Given Kudos: 18
Send PM
Re: Journalist: In physics journals, the number of articles reporting the [#permalink]
1
Kudos
ankitranjan wrote:
Journalist: In physics journals, the number of articles reporting the results of experiments involving particle
accelerators was lower last year than it had been in previous years. Several of the particle accelerators at major
research institutions were out of service the year before last for repairs, so it is likely that the low number of
articles was due to the decline in availability of particle accelerators.
Which of the following, if true, most seriously undermines the journalist’s argument?
(A) Every article based on experiments with particle accelerators that was submitted for publication last year
actually was published.
(B) The average time scientists must wait for access to a particle accelerator has declined over the last
several years.
(C) The number of physics journals was the same last year as in previous years.
(D) Particle accelerators can be used for more than one group of experiments in any given year.
(E) Recent changes in the editorial policies of several physics journals have decreased the likelihood that
articles concerning particle-accelerator research will be accepted for publication.

OA will be given tomorrow.


If You Like the question Consider KUDOS


conclusion : the decrease in number of publication last year was due to unavailability of particle accelerator.
premise 1/ reason : several particle accelerators were out of service


to weaken this argument/conclusion lets look out for an alternate reason leading to conclusion. this is exactly wht option E does, it shows decrease is due to changes in editorial policies.
User avatar
Manager
Manager
Joined: 03 Aug 2011
Posts: 189
Own Kudos [?]: 55 [2]
Given Kudos: 12
Location: United States
Concentration: General Management, Entrepreneurship
GMAT 1: 750 Q49 V44
GPA: 3.38
WE:Engineering (Computer Software)
Send PM
Re: Journalist: In physics journals, the number of articles reporting the [#permalink]
2
Kudos
D- even if particle accelerators can be used by multiple groups, they didn't state how many groups there are and how many particle accelerators there are. perhaps if the ratio changed, then maybe we could consider this, but even in the premises they never excluded the possibility of sharing.

B- this one seemed trickier to exclude.. if the time scientists wait is decreasing, one would assume that it can't be the accelerators being broken causing the lower articles.. ie scientists have even faster access. but what if the fact that the accelerator is broken, many groups don't even bother pursuing their experiment and move onto other things. then they technically have zero wait.

say there are 4 researchers waiting 1 day to use a machine, then say the machines are improved so the researchers only have to wait 1/2 day (ans B), well there are still only 4 articles published, although faster perhaps

with E, if their articles aren't even published, then it shows the wait time has no effect on the number of articles.
Current Student
Joined: 10 Mar 2013
Posts: 360
Own Kudos [?]: 2713 [2]
Given Kudos: 200
Location: Germany
Concentration: Finance, Entrepreneurship
GMAT 1: 580 Q46 V24
GPA: 3.7
WE:Marketing (Telecommunications)
Send PM
Re: Journalist: In physics journals, the number of articles reporting the [#permalink]
2
Kudos
suyash wrote:
What's wrong with D.
Both D and E are equal contenders.Both of them are weakning the argument in their own ways
D is weakening the argument by stating that
"Although the effect occurs, the cause was not the reason for the occurence of the effect"

E is weakening the argument by stating that
"The stated cause was not the reason that the effect occured"

How do you break the tie between the two, as both of them qualify equally in weakening the argument.
I find D more appealing in that, E mentions the word recent, which may mean different for different people.
It may mean this year, or it may mean previous 5 years. Who knows ? :)

Somebody kindly enlighten.


I had also to chose between E&D - I've picked E, because it states an alternative Reason. But I found D is also weakening the argument. I saw your explanation for D and actually at the point realised why D is wrong.

Although the effect occurs, the cause was not the reason for the occurence of the effect the fact is that the effect doesn't occur -->> if 1 accelerator can be used multiple times than you won't have a drop in the number of articles - the effect did not take place in this construcition + the argument is not weakened it just don't exist after D - we don't have neither the effect nor the cause (the availability didn't fall if they can be used mupltiple time)

Explantion from a GMAT Expert on the beatthegmat: The problem with D is that it doesn't address the essence of the problem, which is the difference between this year and previous years.

As stated, D is something that is true for accelerators in general-and that thus has always been true, both in previous years and now. So, it can't explain a difference between this year and previous years.


Update 16.04.15
Second try - had to struggle between B and E:
According to the CR Bible B is wrong:

Structire: Low availability (CAUSE) --> (EFFECT) Low # of articles
Answer B) Says that the availability have not declined - Cause doesn't oocur, so does the effect (if we have a better availability of the accelerators, then the # of articles must also be high)
--> This destroys the whole argument.

A) Find an alternative clause - is not the case here (E plays this role)
B) If cause occurs, the effect doesn't occur - is not the case here
C) Effect occurs, cause did not occur - is not the case here
D) Reversed relationship - is not the case here
E) Statistical problem - is not the case here
Senior Manager
Senior Manager
Joined: 04 Jun 2016
Posts: 484
Own Kudos [?]: 2352 [4]
Given Kudos: 36
GMAT 1: 750 Q49 V43
Send PM
Re: Journalist: In physics journals, the number of articles reporting the [#permalink]
4
Kudos
THE CORRECT ANSWER IS E

Consider this analogy
Last summer hundreds of parrot regularly flew from nearby jungle and came to your garden to drink water from the water-fountain you installed in the corner of the garden
This summer no parrot came to drink water from your water-fountain
Wrong conclusion :- All parrots must be dead thats why they are not coming to drink water.
Right conclusion :- Last year due to draught and no rainfall, the river inside the jungle was dry. This year the river is full of water because of excess rainfall and cooler temperature.

Similarly:- Last year all particle accelerator were not out of service. They were working and lots of experiment were happening and lots of those articles were submitted to journal.
Then why were there less articles about particle physics journals. Because despite being submitted they were not published.
Why were the not published .. because new policies have limited the total number of such articles that can be published.

(E) Recent changes in the editorial policies of several physics journals have decreased the likelihood that articles concerning particle-accelerator research will be accepted for publication.



ankitranjan wrote:
Journalist: In physics journals, the number of articles reporting the results of experiments involving particle
accelerators was lower last year than it had been in previous years. Several of the particle accelerators at major
research institutions were out of service the year before last for repairs, so it is likely that the low number of
articles was due to the decline in availability of particle accelerators.
Which of the following, if true, most seriously undermines the journalist’s argument?
(A) Every article based on experiments with particle accelerators that was submitted for publication last year
actually was published.
(B) The average time scientists must wait for access to a particle accelerator has declined over the last
several years.
(C) The number of physics journals was the same last year as in previous years.
(D) Particle accelerators can be used for more than one group of experiments in any given year.
(E) Recent changes in the editorial policies of several physics journals have decreased the likelihood that
articles concerning particle-accelerator research will be accepted for publication.

OA will be given tomorrow.


If You Like the question Consider KUDOS
Manager
Manager
Joined: 23 Jun 2009
Posts: 128
Own Kudos [?]: 726 [2]
Given Kudos: 138
Location: Brazil
GMAT 1: 470 Q30 V20
GMAT 2: 620 Q42 V33
Send PM
Re: Journalist: In physics journals, the number of articles reporting the [#permalink]
2
Kudos
I disagree with "irrelevant reason" for dropping an answer choice, there must be an especially when we are talking about a couple of grants dollars cost per question from ACT.

Therefore, for every single option, there is a reasoning behind it and somehow it weakens the argument. The point is, which option is fallacy-free, or in other words, which option ALWAYS weaken the argument.

Stem's Conclusion: The low number of articles was due to the decline in availability of particle accelerators.
What happened? Particle accelerators out of service
What happened? #'s of articles was lower last year than it had been in previous years

Always bearing this conjuncture above in mind, I tackle the options

Quote:
(A) Every article based on experiments with particle accelerators that was submitted for publication last year was published.
(B) The average time scientists must wait for access to a particle accelerator has declined over the last
several years.
(C) The number of physics journals was the same last year as in previous years.
(D) Particle accelerators can be used for more than one group of experiments in any given year.
(E) Recent changes in the editorial policies of several physics journals have decreased the likelihood that
articles concerning particle-accelerator research will be accepted for publication.


the reasoning behind each option should be ALWAYS clear.

What are the features that the test maker want me to believe in each of the option?

(A) all submissions published
The test makers want to convince me that a lower number of articles is result from the scarcity of content from the maintenance/shutdown of particle accelerators. However, scientists can submit new research using same data from before the shutdown. Therefore, the same amount of articles is kept, and the given reason cannot ALWAYS weaken the argument

B) Less wait to access the accelerator
The test taker thinks that the fewer scientists wait, the more experiments they do. So, who guarantees that more experiments = more publications? It may be true or not. It can weaken or strengthen the argument, therefore cannot ALWAYS weaken the argument

(C) The number of journals remains unchanged
The test taker thinks that less publication can be affected by the amount of journals available. The idea could be valid unless we presuppose that journals always publish a limited amount of article. Not always weaken the argument, therefore cannot ALWAYS weaken the argument

(D) more than one group use the accelerator in any given year.
The test taker thinks that analysis takes an entire year, even if that is true, other publications with data with other years or conjoint or whatever cannot be published, which is a lie. Therefore cannot ALWAYS weaken the argument

(E) Editorial policies changes of those that publish decreased the likelihood of publication
The test taker assures that the low-end of the publication chain will close the tap, which is a good reason to believe
ALWAYS weaken the argument
avatar
Intern
Intern
Joined: 12 Jul 2017
Posts: 1
Own Kudos [?]: 0 [0]
Given Kudos: 11
Send PM
Re: Journalist: In physics journals, the number of articles reporting the [#permalink]
vineet474 wrote:
Journalist: In physics journals, the number of articles reporting the results of experiments involving particle
accelerators was lower last year than it had been in previous years. Several of the particle accelerators at major
research institutions were out of service the year before last for repairs, so it is likely that the low number of
articles was due to the decline in availability of particle accelerators.
Which of the following, if true, most seriously undermines the journalist’s argument?
(A) Every article based on experiments with particle accelerators that was submitted for publication last year
actually was published.
(B) The average time scientists must wait for access to a particle accelerator has declined over the last
several years.
(C) The number of physics journals was the same last year as in previous years.
(D) Particle accelerators can be used for more than one group of experiments in any given year.
(E) Recent changes in the editorial policies of several physics journals have decreased the likelihood that
articles concerning particle-accelerator research will be accepted for publication.



This is a simple one and you should destroy these questions in less than 1 min and 30 secs.. Here,s how to do it.
By looking at the conclusion and the stem, it is evident that the question is CAUSE AND AFFECT reasoning + WEAKEN question.

So by mapping the conlusion we see that :
PED (decline in availability of particle accelerators) {CAUSE} ---------->>>> LNOA (low number of
articles) {EFFECT}

In these types of questions, following options are to be considered:
A. Find an alternate cause for the stated effect
B. Show that even when the cause occurs, the effect does not occur
C. Show that although the effect occurs, the cause did not occur
D. Show that the stated relationship is in fact reversed
E. Show a statistical problem exists with the data used to make the causal
statement

Mostly answer will be from the first 2 or the last option (which is tricky).
Coming back to the question, find any alternate cause for the stated effect in AC,s. E best explains an alternate cause.

(E) Recent changes in the editorial policies of several physics journals have decreased the likelihood that
articles concerning particle-accelerator research will be accepted for publication.

So, it was not due to the fact that we had less particle accelerators but because of change in
the policy that the number of artiles were decreased. Statement E clearly explain the alternate cause for the stated effect.

HENCE E. Hope it helps. :)

Any suggestions are welcomed.
-V


Still don't understand why (B) and (D) wrong :(
Answer choices (B) & (D) fit the option "although the effect occurs, the cause did not occur"

Will be appreciated, if anyone can explain.
Board of Directors
Joined: 18 Jul 2015
Status:Emory Goizueta Alum
Posts: 3600
Own Kudos [?]: 5442 [5]
Given Kudos: 346
Send PM
Re: Journalist: In physics journals, the number of articles reporting the [#permalink]
4
Kudos
1
Bookmarks
Expert Reply
SuYiChung wrote:
Still don't understand why (B) and (D) wrong :(
Answer choices (B) & (D) fit the option "although the effect occurs, the cause did not occur"

Will be appreciated, if anyone can explain.


Hey SuYiChung ,

You need to understand the cause and effect concept well.

Whenever the effect happens in the past, there are 3 types of weakeners:

1. Alternate cause
2. Reverse Causation
3. Cause not happening before the effect.

Now, here cause is - Decline in the availability of accelerators.

Effect is : Less Particles Published.

Decline in the availability of accelerators --> Less Particles Published.

We need to weaken this, I can say there was another cause for less particles published. This is what option E is doing, hence is the correct answer.

Now, Let's talk about B and D.

B is saying scientists can get the accelerator sooner these days than earlier. It is no where telling us the reason for the decreased publication. Hence, wrong.

Similarly, D is saying you can use the accelerators for multiple experiments. So, what? We don't really know why the publication decreased. Hence, wrong.

Your reasoning "although the effect occurs, the cause did not occur" is wrong here. No effect is occurring in B and D. It is just telling us new information, which is irrelevant.

Does that make sense?
UNC Kenan Flagler Moderator
Joined: 18 Jul 2015
Posts: 238
Own Kudos [?]: 248 [1]
Given Kudos: 120
GMAT 1: 530 Q43 V20
WE:Analyst (Consumer Products)
Send PM
Journalist: In physics journals, the number of articles reporting the [#permalink]
1
Kudos
ankitranjan wrote:
Journalist: In physics journals, the number of articles reporting the results of experiments involving particle accelerators was lower last year than it had been in previous years. Several of the particle accelerators at major research institutions were out of service the year before last for repairs, so it is likely that the low number of articles was due to the decline in availability of particle accelerators.

Which of the following, if true, most seriously undermines the journalist’s argument?

Assume in 2017 the number of articles involving particle accelerators were 20 and in 2018 they were 10, a 50% drop vs. 2017. As per the argument a lot of accelerators were out for maintenance in 2017 and hence not available for research that could have made it to journal citations in 2018.

A couple of weakeners could be:
1. What if all the accelerators came back from repairs within a week itself? And hence it was not the longer turn-around that was responsible for the lower citations
2. What if a lot of scientists were busy with other research initiatives and hence were not available for research involving particle accelerators?
3. What if the experiments were not significant enough or were inaccurate and hence may have got rejected by these publications?


(A) Every article based on experiments with particle accelerators that was submitted for publication last year actually was published.
This attacks the third point from the pre-though examples above and indicates that there were no rejections on the submitted articles. But this option still keeps the possibility open that the maintenance turn-around could be still high because of which sufficient number of experiments could not be made in the first place

(B) The average time scientists must wait for access to a particle accelerator has declined over the last several years.
While this may seem to weaken by stating that the maintenance turn-around has reduced, it fails to quantify the reduction. This option could weaken if it stated that the average time has significantly reduced, whereas on the other hand it would have no impact if there was only a 1% reduction in the turn-around time

(C) The number of physics journals was the same last year as in previous years.
This option has no impact on the argument as we are concerned about the total citations from all journals put together, and not concerned about the articles per journal. Additionally, this option does not talk about the maintenance turn-around time or the availability of scientists

(D) Particle accelerators can be used for more than one group of experiments in any given year.
This option is similar in structure to option A and might also seem to weaken by stating that even though less accelerators were available, scientists could have made the most out of the existing accelerators by conducting more than one group of experiments. But like A this option still keeps the possibility open that because of the high turn-around, by the time scientists received the particle-accelerators it was too late for them to conduct the experiments and meet the deadlines to submit their findings for publication

(E) Recent changes in the editorial policies of several physics journals have decreased the likelihood that articles concerning particle-accelerator research will be accepted for publication.
This clearly introduces an additional reason stating that there are a new set of policies in place which would decrease the chances of articles concerning particle-accelerator to show up in journals


Correct Ans - E
Manager
Manager
Joined: 15 Jun 2019
Posts: 135
Own Kudos [?]: 217 [0]
Given Kudos: 123
Send PM
Re: Journalist: In physics journals, the number of articles reporting the [#permalink]
vineet474 wrote:

In these types of questions, following options are to be considered:
A. Find an alternate cause for the stated effect
B. Show that even when the cause occurs, the effect does not occur
C. Show that although the effect occurs, the cause did not occur
D. Show that the stated relationship is in fact reversed
E. Show a statistical problem exists with the data used to make the causal
statement



Any suggestions are welcomed.
-V


For E, (E. Show a statistical problem exists with the data used to make the causal
statement)
can anybody provide eg to nail that concept??
Senior Manager
Senior Manager
Joined: 28 Jan 2017
Posts: 365
Own Kudos [?]: 79 [0]
Given Kudos: 832
Send PM
Re: Journalist: In physics journals, the number of articles reporting the [#permalink]
Dear VeritasKarishma,

Does the conclusion say that the decline in availability of particle accelerators is the ONLY cause to the effect?
Tutor
Joined: 16 Oct 2010
Posts: 14913
Own Kudos [?]: 65699 [4]
Given Kudos: 431
Location: Pune, India
Send PM
Re: Journalist: In physics journals, the number of articles reporting the [#permalink]
3
Kudos
1
Bookmarks
Expert Reply
varotkorn wrote:
Dear VeritasKarishma,

Does the conclusion say that the decline in availability of particle accelerators is the ONLY cause to the effect?


Every question is worded differently so what works in one may not be right in another. Don't try to club CR questions into distinct strategy buckets and don't try to form general rules for them. The call needs to be taken on question to question basis.

Here the argument says "A happened, perhaps that is why B happened".

Correct option says "C happened and that could have caused B" This weakens our conclusion for why B happened. It is possible that both A and C can cause B but our conclusion points at A being the reason for B this time around. If C happened too this time, that could have been the reason for B or perhaps both caused B. In any case, it decreases the probability that B happened because of A.


The other question on your mind is different. There we had to prove a generic "A causes B". We had to find whether A causes B or not.
That does not depend on the fact whether "C causes B" too.
GMAT Club Bot
Re: Journalist: In physics journals, the number of articles reporting the [#permalink]
 1   2   
Moderators:
GMAT Club Verbal Expert
6954 posts
GMAT Club Verbal Expert
238 posts
CR Forum Moderator
820 posts